Question

A circuit system contains a circular wire with a radius of r=13.5*10^-2m. The circuit has a resister with resistance equaling r1= 9.9 ohms. The circuit is entirely immersed in a uniform magnetic field with a magnitude that changes over the time. If the rate of change of the magnetic field is either decreasing - or increasing + with ΔΒ ΔΙ 6.8 T/s,.х ххк Х Х X x x кxx x->B into page

a) find current induced in circuit. Write answer in mA and to 2 decimal places

b) indicate direction of current

0 0
Add a comment Improve this question Transcribed image text
Answer #1

yr, = 9.92 r=13.58102 m. AB = 608 T/s At Area A = T2 = T (13:58152) 2 A: 5.73X102 m2 Hence euf LE = A dB at at (5.73 x103) x6

Add a comment
Know the answer?
Add Answer to:
A circuit system contains a circular wire with a radius of r=13.5*10^-2m. The circuit has a...
Your Answer:

Post as a guest

Your Name:

What's your source?

Earn Coins

Coins can be redeemed for fabulous gifts.

Not the answer you're looking for? Ask your own homework help question. Our experts will answer your question WITHIN MINUTES for Free.
Similar Homework Help Questions
  • A circuit consists of a circular loop of wire with a radius a=28.3×10^-2 m. the circuit...

    A circuit consists of a circular loop of wire with a radius a=28.3×10^-2 m. the circuit has a resistor with a resistance R1 = 9.1 . the circuit is entirely immersed a uniform magnetic filed whose magnitude changes over time . if the rate of change of the magnetic field is either decreasing or increasimg with 3.7 T/s a)find the current induced in the circuit. write your answer in mA and to 2 decimal places b) indicate the direction of...

  • A circuit consists of a circular loop of wire with a radius a = 35.5 x...

    A circuit consists of a circular loop of wire with a radius a = 35.5 x 102 m. The circuit has a resistor with a resistance R = 1.8 as shown below. The circuit is entirely immersed a uniform magnetic field whose magnitude changes over time. If the rate of change of the magnetic field is either decreasing (-) or increasing (+) with a AB = -9.6 T/s, Ri X At Buto la page X X X X X X...

  • A circular loop of wire of resistance - 0.500 and radius 8.10 cm is in a...

    A circular loop of wire of resistance - 0.500 and radius 8.10 cm is in a uniform magnetic field directed out of the page as in the figure below. A clockwise current of 120 mA is induced in the loop. (a) is the magnetic field increasing or decreasing in time? decreasing (b) Find the rate at which the field is changing with time.

  • Q1. (25 points) A circular loop of wire of resistance R = 1 N and radius...

    Q1. (25 points) A circular loop of wire of resistance R = 1 N and radius r = 15 cm is in a uniform magnetic field directed out of the page as shown in the figure. a) (7 points) If a clockwise current of I = 8 mA is induced in the loop, is the magnetic field increasing or decreasing in time? b) (8 points) Find the induced emf in the loop. c) (10 points) Find the rate at which...

  • Q1. (25 points) A circular loop of wire of resistance R = 4 N and radius...

    Q1. (25 points) A circular loop of wire of resistance R = 4 N and radius r = 30 cm is in a uniform magnetic field directed out of the page as shown in the figure. a) (7 points) If a clockwise current of 1 = 20 mA is induced in the loop, is the magnetic field increasing or decreasing in time? b) (8 points) Find the induced emf in the loop. c) (10 points) Find the rate at which...

  • 3. Consider a circular loop of a wire with a radius of r = 20.0 cm...

    3. Consider a circular loop of a wire with a radius of r = 20.0 cm in a uniform magnetic field of B = 0.300 T pointing into the page, as shown below. The loop of wire has a resistance of 2.5 12. xx x x x x X X X X х X х X X X x x х х х X B х XX x x r XX X X х х x x X X X...

  • A circular loop of wire of resistance 0.4 Ohm and radius 11 cm, sitting n the...

    A circular loop of wire of resistance 0.4 Ohm and radius 11 cm, sitting n the plane of this paper, is in a uniform magnetic field, directed out of the paper. A counterclockwise induced current of 3.5 mA is measured in the loop. Is the magnetic field increasing or decreasing as time elapses? Explain how you know. At what rate is the magnetic field changing with time?

  • Problem 1 (20 points] A circular loop of wire with radius r = 10 cm and...

    Problem 1 (20 points] A circular loop of wire with radius r = 10 cm and Resistance R = 1 N is * in a region of uniform magnetic field, as shown in the figure. The magnetic field is directed into the plane. At t = 0s, the magnetic field * * is zero. Then, the magnetic field starts to increase as function of time, B(t) = 0.5t? * * * X X a) [5 points) is the magnetic flux...

  • 3. Consider a circular loop of a wire with a radius of r = 20.0 cm...

    3. Consider a circular loop of a wire with a radius of r = 20.0 cm in a uniform magnetic field of B = 0.300 T pointing into the page, as shown below. The loop of wire has a resistance of 2.5 2 X X X X X X X X X X X X X x x x В x x x x x x x x x X XX Х X/X X X X x x x x...

  • The figure below shows a circular loop of wire of resistance R = 0.500Ω and radius r = 9.30 cm in the presence of a uniform magnetic field B out directed out of the page

    The figure below shows a circular loop of wire of resistance R = 0.500Ω and radius r = 9.30 cm in the presence of a uniform magnetic field B out directed out of the page. A clockwise current of I = 3.30 mA is induced in the loop.(a) Which of the following best describes the magnitude of Bout It is increasing with time. It is decreasing with time. It remains constant. (b) Find the rate at which the field is changing with time (in...

ADVERTISEMENT
Free Homework Help App
Download From Google Play
Scan Your Homework
to Get Instant Free Answers
Need Online Homework Help?
Ask a Question
Get Answers For Free
Most questions answered within 3 hours.
ADVERTISEMENT
ADVERTISEMENT
ADVERTISEMENT